solve for y 7y - 6y - 10 = 13

Answers

Answer 1

Answer: y=23

Step-by-step explanation:

[tex]7y-6y-10=13[/tex]

[tex]7y-6y=y[/tex]

[tex]y-10=13[/tex]

add 10 to both sides

[tex]y-10+10=13+10[/tex]

[tex]y=23[/tex]


Related Questions

Whats the volume of a ball if diameter is 17

Answers

Answer:

818.8333333 *pi

or

Using the pi button on the calculator

2572.440785

Using 3.14 for pi

2571.13666666

Step-by-step explanation:

We need the radius

r = d/2

r = 17/2 = 8.5

We can find the volume using

V = 4/3 pi r^3

V = 4/3 pi ( 8.5)^3

V =818.8333333 *pi

  Using the pi button on the calculator

2572.440785

Using 3.14 for pi

2571.13666666

You move at 45/mph and a bullet is shot at you, the bullet is moving at 44/mph

Would you be able to dodge the bullet, you factor in the fight or flight or freeze response

Answers

Answer:

See explanation below.

Step-by-step explanation:

A bullet shot at you will definitely hit you because of the direction it is travelling.

Your fight and flight response won't be activated either because of the speed of the event (bullet) or if you don't know how harmful a gun is (you'll remain calm/ignorant in this case)?

Try to post academic problems.

Best Regards!

A set of furniture was sold for GH cedis 3000 at a profit of 25%. Find the cost price.

Answers

Answer:

2400GH Cedis

Step-by-step explanation:

[tex]Selling \:price = 3000gh Cedis\\Profit \% = 25\%\\Cost\:price =?\\\\CP =\frac{ ( Selling\:price \times 100 )}{ ( 100 + percentage profit)} \\\\C.P = \frac{3000\times 100 }{100+25}\\\\ C.P = \frac{300000}{125} \\\\Cost\: Price = 2400gh Cedis[/tex]

Hey there please help me with this question

Answers

Answer:

see explanation

Step-by-step explanation:

sum the parts of the ratio, 2 + 1 = 3 parts , thus

81 cm² ÷ 3 = 27 cm² ← value of 1 part of the ratio

2 parts = 2 × 27 = 54 cm²

Area of A = 54 cm² and area of B = 27 cm²

The side of the original square = [tex]\sqrt{81}[/tex] = 9 cm

The width of both rectangles is 9 cm ( width remains unchanged after cut )

Thus

Rectangle A

9 × length = 54 ( divide both sides by 9 )

length = 6 cm

Rectangle B

9 × length = 27 ( divide both sides by 9 )

length = 3 cm

Rectangle A → length = 6 cm, width = 9 cm

Rectangle B → length = 3 cm , width = 9 cm

Answer:

Rectangle A                 Rectangle B

length = 9 cm                length = 9 cm

width = 6 cm                  width = 3 cm

Step-by-step explanation:

Area of square At = 81 cm²

Square is cut into two pieces = A + B

The ration of area A to B = 2:1

Find

Rect A        Rect B

length         length

width           width

---------------------------------

first, get the side of the square = A = s²

81 = s²,      

s = √81      

s = 9 cm

since the ratio is 2:1, therefore the side can be divided into 3

9 ÷ 3 = 3 cm ----- take note of this to get the Width

Rectangle A

L = 9 cm (which is the s = 9 cm)

W = 3 cm (2 ratio) = 6 cm

Rectangle B

L = 9 cm  (which is the s = 9 cm)

W = 3 cm (1 ratio) = 3 cm

Proof:

At = A + B

81 = (9x6) + (9x3)

81 = 54 + 27

81 = 81  ----- OK

Do the ratios 2/3 and 12/18 form a proportion?
yes
no​

Answers

Answer:

Yes

Step-by-step explanation:

Because 12/18 = 2/3..(cancel 12 and 18 by 6)

Answer:

yes

Step-by-step explanation:

2x6=12

3x6=18

6 is the multiplying number

( the 2 equations are the same amount )

The fraction subtracted from 5/3 to get 1 is_____​

Answers

Answer:

2/3

Step-by-step explanation:

I am not sure

Answer:

2/3

Step-by-step explanation:

[tex]Let \:the \: unknown \: fraction \: be \: x\\\\\frac{5}{3} -x = 1\\\\\frac{5}{3}-x=1\\\\\mathrm{Subtract\:}\frac{5}{3}\mathrm{\:from\:both\:sides}\\\\\frac{5}{3}-x-\frac{5}{3}=1-\frac{5}{3}\\\\\frac{5}{3}-x-\frac{5}{3}=-x\\\\1-\frac{5}{3}=-\frac{2}{3}\\-x=-\frac{2}{3}\\\\x=\frac{2}{3}\\[/tex]

what is the square root of 450​

Answers

Answer:

[tex]15\sqrt{2}[/tex]      or       21.2132

Step-by-step explanation:

[tex]\sqrt{450}[/tex]

[tex]\sqrt{15^{2} }[/tex]   (root of a product is equal to the product of the roots of each factor)

[tex]\sqrt{15^{2} } \sqrt{2}[/tex]                (simplify)

[tex]15\sqrt{2}[/tex]    or   ≈ 21.2132

Answer:

[tex]15\sqrt{2}[/tex] or 21.213

Step-by-step explanation:

For radical form: think of multiples of 450. Think of a pair that contains one perfect square, particularly the higher, the better . These 2 numbers are 25 and 18. 25 is the perfect square number since the two numbers that multiply to be 25 is 5 and 5.

Now take the perfect square of 25 and put it outside of the radical. The 18 remains inside: [tex]5\sqrt{18}[/tex]

Now, since 18 is a high number that needs to get reduced, do the same for 18 as we did for 450--find two numbers, one of which is a perfect square. These two numbers are 9 and 2.

Now take the perfect square of 9. This is 3. Take it out of the radical so that only the two remains inside. The 3 will now multiply with the 5: [tex]5*3\sqrt{2}[/tex]

Multiply 5 and 3 to get 15. The 15 stays outside the radical. Your answer is:

[tex]15\sqrt{2}[/tex]

look at the image to get ur question thank u for ansering

Answers

Answer:

y = 3x + 3

Step-by-step explanation:

The equation of a line in slope- intercept form is

y = mx + c ( m is the slope and c the y- intercept )

Calculate the slope using the slope formula

m = [tex]\frac{y_{2}-y_{1} }{x_{2}-x_{1} }[/tex]

with (x₁, y₁ ) = (0, 3) and (x₂, y₂ ) = (- 1, 0) ← 2 points on the line

m = [tex]\frac{0-3}{-1-0}[/tex] = [tex]\frac{-3}{-1}[/tex] = 3

The line crosses the y- axis at (0, 3) ⇒ c = 3

y = 3x + 3 ← equation of line

Mr.Snyder gave his four children $35 to split equally for each car they cleaned out. The children cleaned put 3 cars. Mr.Snyder does not have any coins. He only had dollar bills. How much money should each child get?

Answers

Answer:

$26 for each child

Step-by-step explanation:

35 * 3 = 105 / 4 = 26.25

i just answered this, they would each get $26

Write an expression that can be used to find the price of a television that is on sale for 20% off the regular price of p dollars. Can you write a second expression equivalent to the one you wrote in the last questions.

Answers

Answer:

The expression that could help calculate the price of the TV is;

$P - 20% of $P

Step-by-step explanation:

Here, we want to write an expression that corresponds to the price of a television set that is on sale at a price which is 20% off the regular price.

From the question, we can see that the regular price is $P

So now we are having 20% off;

This corresponds to;

20/100 * p = p/5 = 0.2p

So in the expression form, we can have;

$P - 20% of $P

What else would need to be congruent to show that AABC = APQR by SSS?
Giver
AC =PR
BC = QR

A AC = PQ
B. AB = PQ
C. ZAZP
D. ABBA

Answers

Answer:

B, because that's the side that wasn't given

Step-by-step explanation:

The statement AB ≅ PQ is required to be congruent.

Option B is the correct answer.

What is triangle congruency?

There are ways to prove that two triangles are congruent.

- Side-Side-Side (SSS) Congruence.

The three sides of one triangle are equal to the corresponding three sides of another triangle.

- Side-Angle-Side (SAS) Congruence.

The two sides and the included angle of one triangle are equal to the corresponding two sides and included angle of another triangle.

- Angle-Side-Angle (ASA) Congruence.

The two angles and the included side of one triangle are equal to the corresponding two angles and included side of another triangle.

- Angle-Angle-Side (AAS) Congruence.

We have,

Side-Side-Side (SSS) Congruence.

The three sides of one triangle are equal to the corresponding three sides of another triangle.

Now,

ΔABC and ΔPQR

AC ≅ PR  (given) (side)

BC ≅ QR (given ) (side)

AB ≅ PQ (side)

ΔABC ≅ ΔPQR (SSS)

Thus,

AB ≅ PQ is required to be congruent.

Learn more about triangle congruency here:

https://brainly.com/question/12413243

#SPJ7

Double a number decreased by 25.6 is equal to 90 Find the number

Answers

Answer:

Step-by-step explanation:

2x-25,6=90

2x=90+25,6

2x=115,6

x=57,8

Answer:

The number is 57.8

Step-by-step explanation:

Let x = number

2x -25.6 = 90

Add 25.6 to each side

2x-25.6 +25.6 = 90+25.6

2x=115.6

Divide by 2

2x/2 =115.6/2

x =57.8

What’s the difference between rational and irrational numbers?

Answers

Answer:

rational numbers are perfect squares irrational numbers are non terminating/go on forever

Step-by-step explanation:

A woman sold an article for 200 GH cedis and made a profit of 25%. Find the cost price of the article.

Answers

Answer:

160

Step-by-step explanation:

x = original price

x+ 25% x = 200

Change to decimal form

x + .25x = 200

1.25x = 200

Divide each side by 1.25

x = 200/1.25

x =160

The original price was 160

WILL GIVE BRAINLIEST

Answers

intercept is the length from origin to intersection point of respective axis,

it intersects x axis at -7.5 and y is 0 so x intercept is (-7.5,0)

and similarly, y intercept is (0,5.5)

Answer:

(- 7.5, 0 ) and (0, 5.5 )

Step-by-step explanation:

The x- intercept is the value of x on the x- axis where the line crosses.

Here the line crosses the x- axis at - 7.5 , thus the coordinates of x- intercept

(- 7.5, 0 )

The y- intercept is the value of y on the y- axis where the line crosses.

Here the line crosses the y- axis at 5.5 thus the coordinates of the y- intercept

(0, 5.5 )

In the expression 3x^2+y+-5 which of the following choices is the exponent in the term 3x^2?

A. 3

B. 2

C. X

D. None of these choices

Answers

Answer:

2

Step-by-step explanation:

3x^2

The coefficient is 3

The variable is x

The exponent is 2

if a man works 400km in 6 minutes.How long will he work in 9 minutes​

Answers

Answer:

600 km

Step-by-step explanation:

400 km  =       x    

  6 min         9 min  

cross multiply:

6x = 400 ( 9)

x = 3600 / 6

x = 600 km

What's the correct answer to this..? Need help

Answers

Answer:

A.

Step-by-step explanation:

All of those graphs represent functions. You are able to tell because they all pass the vertical line test. The vertical line test is conducted by drawing a line that goes vertically and intercepts any point of the line in question. If the function crosses the vertical line twice, it is not a function. If it only intercepts once, it is a function. In this case, every graph is a function because they would intercept a vertical line once.  

Determine if the ordered pair (6, 4) is a solution to the inequality

Answers

Answer:

[tex]\Large \boxed{\mathrm{Option \ D}}[/tex]

Step-by-step explanation:

(6, 4)

x = 6 and y = 4

y > -1/2x + 7

Plug in the values to check if it is true.

4 > -1/2(6) + 7

4 > -3 + 7

4 > 4

This statement is false.

(6, 4) lies on the line.

help help help help help help help help help help help help help help help help help help help help help help help help help help help help help help help help help help help help help help help help help help help help help help help help help help help help help help help help help help help help help help help help help help help help ill give brainly ;)))

Answers

Answer:

A: 2:30

B: 13:15.

C: 4:05 A.M

D: 3:25 P.M

Step-by-step explanation:

Answer:

a) 0230 hours

b) 1315 hours

c) 4:05 am

d) 3:25 pm

Step-by-step explanation:

To change a 12 hour clock to a 24 hour clock, note that when it hits p.m., anything over 12 p.m. will have the number added to it.

a) 2:30 am

2:30 am is earlier than 12 pm, therefore, 0230 is your answer. Remember, you must place the 0 in the front to not confuse a user that the 2 is in the tens place value.* *Essentially, use military time:

b) 1:15 pm

1:15 pm is later than 12 pm, therefore, add 12 to the 1 in the hours. Use military time:

0115 + 1200 = 1315

1315 hours is your answer.

To change a 24 hour clock to a 12 hour clock, note the time frame in which the time is given. if over 1200, then subtract by 12, and add the p.m.

0405 = 4:05 am.

1525 = 12 pm + 3:25 = 3:25 pm.

~

A student earned grades of ​, ​, ​, ​, and . Those courses had the corresponding numbers of credit hours ​, ​, ​, ​, and . The grading system assigns quality points to letter grades as​ follows: A​4; B​3; C​2; D​1; F 0. Compute the grade point average​ (GPA) as a weighted mean and round the result with two decimal places. If the​ Dean's list requires a GPA of 3.00 or​ greater, did this student make the​ Dean's list

Answers

Answer:

grade point average​ (GPA) as a weighted mean = 3.14

Yes, student makes the Dean's list.

Step-by-step explanation:

Since the data is not given I will explain the question with a relevant example:

For example

Data:

Students grades are:  A, C, B, A, D

The corresponding number of credit hours: 3, 3, 3, 4, 1

The grading system assigns quality points to letter grades as​:

A = 4

B = 3

C = 2

D = 1

F = 0

To find:

grade point average​ (GPA) as a weighted mean

If the​ Dean's list requires a GPA of 3.00 or​ greater, did this student make the​ Dean's list?

Solution:

Weighted Mean =   Σx[tex]_{i}[/tex] w[tex]_{i}[/tex] / Σw[tex]_{i}[/tex]

Here

Using the earned grades of A, C, B, A, D and corresponding quality points to these letter grades we get:

A = 4

C = 2

B = 3

A = 4

D = 1

So the values in x[tex]_{i}[/tex] are:

x

4

2

3

4

1

Now the number of credit hours are represented as weights w[tex]_{i}[/tex] :

w

3

3

3

4

1

In order to calculate weighted mean first multiply x[tex]_{i}[/tex] with w[tex]_{i}[/tex]

x[tex]_{i}[/tex] w[tex]_{i}[/tex]

4 * 3 = 12

2 * 3 = 6

3 * 3 = 9

4 * 4 = 16

1 * 1 = 1

The sum of x[tex]_{i}[/tex] w[tex]_{i}[/tex] is :

Σx[tex]_{i}[/tex] w[tex]_{i}[/tex] = 12 + 6 + 9 + 16 + 1 = 44

Σx[tex]_{i}[/tex] w[tex]_{i}[/tex]  =  44

Now compute the sum of w[tex]_{i}[/tex]

Σw[tex]_{i}[/tex] = 3 + 3 + 3 + 4 + 1 = 14

Σw[tex]_{i}[/tex] = 14

Putting the values in the weighted mean formula:

Weighted Mean =   Σx[tex]_{i}[/tex] w[tex]_{i}[/tex] / Σw[tex]_{i}[/tex]

                           =  44 / 14

                           = 3.1429

Weighted Mean =   Σx[tex]_{i}[/tex] w[tex]_{i}[/tex] / Σw[tex]_{i}[/tex] = 3.14

Since the​ Dean's list requires a GPA of 3.00 or​ greater and the grade point average​ (GPA) as a weighted mean of the student is 3.14 so the student makes the​ Dean's list because his/her GPA is higher than 3.00

I am unable to find the answer to this pleas help me

Answers

Answer:

Step-by-step explanation:

[tex]\frac{1}{4}[/tex] of 600 is 150

hope this helps

plz mark as brainliest!!!!!!

Rodrigo and his cousin have 8 bags of marbles. Some of the bags contain 5 white marbles. The rest of the bags contain 7 pink marbles. They have 48 marbles in all. How many bags of each color marble do the cousins have?

Answers

Answer:

The cousins have 4 bags of white marbles and 4 bags of pink marbles.

Step-by-step explanation:

If x is the number of bags with white marbles and y is the number of bags with pink marbles, we can write the following system:

x + y = 8 -- Equation 1

5x + 7y = 48 -- Equation 2

5x + 5y = 40 -- Equation 3 (Multiply Equation 1 by 5)

2y = 8 -- (Subtract Equation 3 from 2)

y = 4 -- (Divide both sides by 2)

x + 4 = 8 -- (Substitute y = 4 into Equation 1)

x = 4 -- (Subtract 4 from both sides)

inscribed angles. help asap!​

Answers

Answer:

20°

Step-by-step explanation:

The measure of the inscribed angle is equal to the half of the arc it sees

Since AC is the diameter the measure of arc ABC is 180°

and since A sees arc BC and C sees the arc AB

A< + C< = 90° so angle C = 20°

*This Question Has 3 Questions in it
Question 1 Use a calculator to find [tex]\sqrt{61.6}[/tex] to the nearest hundredth.
Question 2 Which of the following statements is true?
Refer to Question 2 . PNG

Question 3 simplify [tex]\sqrt{0.81}[/tex]

Answers

Answer:

Ques 1: 7.85

Ques 2: both [tex]\sqrt{0.25}[/tex] and [tex]-\sqrt{16}[/tex] are rational.

Ques 3: 0.9

Step-by-step explanation:

Ques 1:

Solving the [tex]\sqrt{61.6}[/tex], we get 7.8485.

Rounding off to nearest hundredth, we see that next digit is 8, so we increase the previous digit by 1.

So, the answer to nearest hundredth is 7.85.

Ques 2:

[tex]\sqrt{0.25}[/tex] and [tex]-\sqrt{16}[/tex]to be checked  whether they are rational or irrational.

We know that [tex]\sqrt{16}[/tex] is equal to 4.

[tex]-\sqrt{16}[/tex] = -4 which is a rational value.

Let us solve [tex]\sqrt{0.25}[/tex] now.

[tex]\sqrt{0.25} = \sqrt{\dfrac{25}{100}}\\\Rightarrow \dfrac{5}{10}[/tex]

Which is a rational value.

So, both are rational.

Ques 3:

Simplify [tex]\sqrt{0.81}[/tex]

0.81 can be written as [tex]\frac{81}{100}[/tex] in rational form.

Now, taking the square root, we need to take the square root for both Numerator and Denominator and then we can divide to get the desired square root.

[tex]\sqrt{0.81} = \sqrt{\dfrac{81}{100}}\\\Rightarrow \dfrac{9}{10} = \bold{0.9}[/tex]

So, the answers are:

Ques 1: 7.85

Ques 2: both [tex]\sqrt{0.25}[/tex] and [tex]-\sqrt{16}[/tex] are rational.

Ques 3: 0.9

Donny has three times as many candy canes as Marc. Marc has thirty more candy canes than Bob. They have 500 candy canes altogether. How many candy canes does Donny have?

Answers

Answer:

318

Step-by-step explanation:

Bob=x

Marc=x+30

Donny=3(x+30)

x+x+30+3x+90=500

5x=500-120

5x=380

x=76

Bob has x = 76

Marc has x+30=106

Donny has 3*112=318

318+106+76=500

if the side length of a square can be represented by 4x + 4 and its area is 1024 square units, find the value of x

Answers

Answer:

x = 7

Step-by-step explanation:

Since it’s the area of a square, we can simply do square root of 1024. (Because to get area of square you do side x side). Which is 32.

So basically 4x + 4 = 32... x = 7

Answer:

x = 7

Step-by-step explanation:

A = 1024

side length of a square = 4x + 4

A = s²

s = √A

s = √1024

s = 32

using the side length to get the value of x

s = 32

4x + 4 = 32

4x = 32 - 4

x = 28 / 4

x = 7

check:

A = side length * side length

A = (4x + 4) * (4x + 4)

A = (4*7 + 4) * (4*7 + 4)

A = 32 * 32

A = 1024  ok

A radioactive substance decays exponentially. A scientist begins with 350 milligrams of a radioactive substance. After 14 hours, 175 mg of the substance remains. How many milligrams will remain after 20 hours

Answers

Answer:

N(t)=N∗.5t/h where n(t) is amount of substance left, N = initial amount, t = current time, and h = half-life time.h = 24 hours t= 45 hours, N = 130mg

therefore, N(t)=130∗.545/24=35.44mg

Step-by-step explanation:

N(t)=N∗.5t/h where n(t) is amount of substance left, N = initial amount, t = current time, and h = half-life time.h = 24 hours t= 45 hours, N = 130mg

therefore, N(t)=130∗.545/24=35.44mg

Answer:

≈ 130 mg

Step-by-step explanation:

This is about the half-life of the substance.

There is a formula for this kind of calculations:

N(t)= N₀*(0.5)^(t/T), where

N(t) = substance left after time period of t,t = time passed,N₀ = initial amount of the substance,T = hal-life time of the given substance.

In our case, we have:

N₀ = 350 mg,t= 20 hours,T = 14 hours  as half of substance decays during this time period,

And the calculation:

N(20)= 350*(0.5)^(20/14)N(20) ≈ 130 mg

Answer: about 130 mg of substance remains after 20 hours

3. If the coordinates of the two points 1 point
are P(-7,5) and Q (-6, 9), then
(abscissa of P) - (abscissa of Q) is.... *
a) –3
O b) 1
c) -2
O d) -1
Other:​

Answers

The answer would have to be a

Please answer answer question now

Answers

Answer:

Area= 97.9 km²

Step-by-step explanation:

Angle at w = 180-119-34

Angle at w = 27°

Side XV= x

x/sin27= 26/sin119

x= sin27(26/sin119)

x= 0.454(26/0.8746)

x= 13.5 km

Side WX = y

y/sin 34= 26/sin119

y= sin34(26/sin119)

y= 0.559(26/0.8746)

y= 16.6 km

S= (16.6+13.5+26)/2

S= 56.1/2

S=28.05

Area = √(28.05(28.05-26)(28.05-13.5)(28.05-16.6))

Area=√(28.05(2.05)(14.55)(11.45))

Area=√(9579.772744)

Area=97.8763

Area= 97.9 km²

Other Questions
How does Bradstreet use figurative language throughout "To My Dear Loving Husband" to develop the theme? Her comparisons emphasize the importance of love. Her comparisons emphasize the reason for love. Her comparisons emphasize her husband's goodness. Her comparisons emphasize her undying love for her husband. Can Someone please help - Write a diary about a person whose depressed and suicidal Moderate drinkers may get some health benefits from alcohol consumption in the form of reduced risks for certain health problems. Which of the following disease risks is NOT improved by moderate alcohol consumption? I need help please, m bda = And m bca = What is the initial temperature (C) of a system that has the pressure decreased by 10 times while the volume increased by 5 times with a final temperature of -123C? please help me solve Please tell the answer define factors affecting enzyme action:temperature [tex] \frac{ {9x}^{2} - {(x}^{2} - 4) {}^{2} }{4 + 3x - {x}^{2} } [/tex]pls help me need help asap For all x, 5-3(x-4)=? Which of the following is an example of an oligopolistic market with a standardized product?A) The market for breakfast cereal.B) The market for aluminum.C) The market for jewelry.D) The market for automobiles. What would be someone experience after being immigrated to America? Choose the option that will correctly complete the sentence bellow. The singer along with a few others .......... The harmonica on stage. A. Play. B. Plays The letters x and y represent rectangular coordinates. Write the given equation using polar coordinates (r,) . Select the correct equation in polar coordinates below.x2+y24x=0a. r=4sinb.r=4cosc.rcos2=4sind.rsin2=4cos The accounts receivable turnover is computed as __________ divided by __________. sales; accounts receivable sales; average accounts receivable sales; net income accounts receivable; net income The first stanza of On Being Brought from Africa, introduces a paradox. Using evidence from the poem, describe the paradox. The ratio of two numbers is 2:3 and the sum of their cubes is 945,what are the two numbers. let the 1st no be=2x and 2nd=3x (2x)^3 + (3x)^3=945 The mole is a counting number that allows scientists to describe how individual molecules and atoms react. If one mole of atoms or molecules is equal to 6.022 x 10^32 atoms or molecules, how many molecules are in 23.45 g sample of copper (II) hydroxide, Cu(OH)2? Express your answer to the correct number of significant figures. (MM of Cu(OH)2 is 97.562g/mol. Be sure to show all steps completed to arrive at the answer. Determine the molar solubility of AgBr in a solution containing 0.150 M NaBr. Ksp (AgBr) = 7.7 10-13. Which of the following will cause an induced current in a coil of wire? A. A wire carrying a constant current near the coil B. A magnet being moved into or out of the coil C. The constant field of the Earth passing through the stationary coil D. A magnet stationary resting near the coil